0 Daumen
584 Aufrufe

Wir betrachten die Funktion f (x) = sin(x)e
x
.
(a) Berechnen Sie das Taylorpolynom dritten Grades der Funktion f in x0 = 0.
(b) Geben Sie an, wie im Falle von Aufgabenteil (a) das Restglied aussieht. (Sie müssen hierzu den Punkt ξ nicht exakt
bestimmen)
(c) Geben Sie mit Hilfe des Satzes von Taylor an, wie groß der Approximationsfehler |f (x) − T3,f
(x; 0)| höchstens
werden kann, wenn wir die Funktion f nur auf dem Intervall
i.
0, π
2

ii.
0, π
6

betrachten.
(d) Zeichnen oder plotten Sie die Funktion f und das Taylorpolynom dritten Grades im Interval [0,π] in ein Koordinatensystem
und legen sie die Zeichnung bzw. den Plot Ihrer Abgabe bei.
(e) Welcher Zusammenhang ist zwischen der Güte der Approximation der Funktion f durch das Taylorpolyom
T3,f
(x; x0
) im Punkt x und dem Abstand von x und x0 erkennbar? (Hier genügt eine Intuition, ein Beweis ist
nicht nötig.)Bildschirmfoto 2018-06-04 um 20.23.03.png

Avatar von

2 Antworten

+1 Daumen
 
Beste Antwort

a)

Bilde alle Ableitung des Polynom dritten Grades:

f(x)=ax^3+bx^2+cx+d

f'(x)=3ax^2+2bx+c

f''(x)=6ax+2b

f'''(x)=6a

Bilde bis zur dritten Ableitung alle von \(f(x)=sin(x)\cdot e^x\):

f'(x)=e^x(sin(x)+cos(x))

f''(x)=2e^x*cos(x)

f'''(x)=-2e^x(sin(x)-cos(x))

Nun kommen wir zum Teil des Bestimmens:

f(x)=ax^3+bx^2+cx+d =sin(0)*e^0 → d=0

f'(x)=3ax^2+2bx+c = e^0(sin(0)+cos(0)) → c=1

f''(x)=6ax+2b = 2e^0*cos(0) → b=2

f'''(x)=6a =-2e^0(sin(0)-cos(0))

6a=2   |:6

a=1/3

Einsetzen:

f(x)=(1/3)x^3+2x^2+x

Hoffe, dass ist richtig und dass ich dir helfen konnte

Avatar von 28 k

Kleiner Fehler. Es muss

$$ f(x)\approx T_3(x;0)=\frac{1}{3}x^3+x^2+x $$

heißen, denn du approximierst ja nur f.

Verstehe ich nicht, aber das macht mit dem Graph mehr sinn.


Einfacher wäre vielleicht \(\left(x-\frac16x^3\right)\cdot\left(1+x+\frac12x^2+\frac16x^3\right)\) auszumultiplizieren.

@ racine_carrée

Ich meinte, dass du f nicht einfach für das Taylorpolynom nehmen darfst. Zum einen ist f(x)=sin(x)*e^x und zum anderen stimmt das Taylorpolynom nur ungefähr.

So tief bin ich nicht in der Materie drin, hab das vor einem Monat ungefähr in nem Artikel gelesen...

0 Daumen

Nach der Restgliedabschätzung von Lagrange hat man bei b)

$$ |R_n(x)|\leq\Bigg|\frac{f^{n+1}(\xi)}{(n+1)!}\cdot x^{n+1} \Bigg| $$Also

$$ |R_3(x)|=\Bigg|\frac{f^{4}(\xi)}{4!}\cdot x^4 \Bigg|=\Bigg|\frac{-4\sin(\xi)e^\xi}{4!}\cdot x^4 \Bigg|\leq\Bigg|\frac{\sin(\xi)e^\xi}{3!}\cdot x^4 \Bigg|\leq\Bigg|\frac{\sin(\xi)e^\xi}{6}\cdot x^4 \Bigg| $$

c) Für (i)

$$|R_3(x)|\leq\Bigg|\frac{\sin(\xi)e^\xi}{6}\cdot x^4 \Bigg|\leq \Bigg|\frac{\sin(\frac{\pi}{2})e^\frac{\pi}{2}}{6}\cdot \Big(\frac{\pi}{2}\Big)^4 \Bigg|=\Bigg|\frac{1\cdot e^\frac{\pi}{2}}{6}\cdot \Big(\frac{\pi}{2}\Big)^4 \Bigg|\\=\frac{e^\frac{\pi}{2}}{6}\cdot \Big(\frac{\pi}{2}\Big)^4<\frac{6}{6}\cdot \Big(\frac{\pi}{2}\Big)^4=\Big(\frac{\pi}{2}\Big)^4 $$

Für (ii)

$$|R_3(x)|\leq\Bigg|\frac{\sin(\xi)e^\xi}{6}\cdot x^4 \Bigg|\leq \Bigg|\frac{\sin(\frac{\pi}{6})e^\frac{\pi}{6}}{6}\cdot \Big(\frac{\pi}{6}\Big)^4 \Bigg|=\Bigg|\frac{0.5\cdot e^\frac{\pi}{6}}{6}\cdot \Big(\frac{\pi}{6}\Big)^4 \Bigg|\\=\frac{e^\frac{\pi}{6}}{2\cdot 6}\cdot \Big(\frac{\pi}{6}\Big)^4<\frac{1}{6}\cdot \Big(\frac{\pi}{6}\Big)^4=\frac{\pi^4}{6^5} $$

Avatar von 14 k

Ein anderes Problem?

Stell deine Frage

Willkommen bei der Mathelounge! Stell deine Frage einfach und kostenlos

x
Made by a lovely community